千葉大学(理系)2019年前期 問題1

$ a_1=3,a_2=2\ とし、n \geqq 2 \ のとき、a_{n+1}=a_n ^2 + a_n -1 \ として数列\{a_n\}を定める。$
$\qquad (1) \quad n \geqq 2 \ のとき \ \ a_{n+1}=a_1a_2 \cdots a_n -1 \ \ が成り立つことを証明せよ。$
\[(2) \quad \sum _{i=1}^n a_i ^2=a_1a_2 \cdots a_n +100 \ \ が成り立つような自然数nを求めよ。\]

$(1)は数学的帰納法をつかうかまたは漸化式をうまく処理します。$
$\qquad ちょっとした落とし穴がありますので注意が必要です。$
$(2)は(1)を利用すれば比較的簡単に求まります。$
$おまけとして与えられた漸化式を拡張します。(これがこの原稿を書いた目的です)$

(1)数学的帰納法で示します。


(i)$\ \ n=2 のとき$
$\quad 左辺=a_3=a_2^2+a_2-1=4+2-1=5$
$\quad 右辺=a_1a_2-1=3 \times 2 -1=5$
$よって n=2 のとき成りたつ。$

(ii)$\ \ n=k \ \ (k \geqq 2)のとき成りたつとすると$
$\qquad a_{k+1}=a_1a_2 \cdots a_k -1 $

$このとき$
\begin{eqnarray*} a_{k+2} &=&a_{k+1}^2 + a_{k+1} -1\\ &=&(a_{k+1}+1) a_{k+1} -1\\ &=&(a_1a_2 \cdots a_k) a_{k+1} -1\\ &=&a_1a_2 \cdots a_k a_{k+1} -1\\ \end{eqnarray*} $よって n=k+1 \ のときも成りたつ。$

(i)(ii)$より \ n \geqq 2 \ のすべての自然数nで a_{n+1}=a_1a_2 \cdots a_n -1 \ \ が成り立つ。$


(2)

$n \geqq 2 \ \ のとき、a_{n+1}=a_n ^2 + a_n -1 \ \ より$

$\qquad a_n ^2 =a_{n+1}- a_n +1 $

$nに2,3,\cdots を代入して$

$\qquad a_2 ^2 =a_3- a_2 +1 $
$\qquad a_3 ^2 =a_4- a_3 +1 $
$\hspace{4em} \vdots $
$\qquad a_n ^2 =a_{n+1}- a_n +1 $

$これらを加えると$

$\qquad a_2^2+a_3 ^2 +\cdots a_n ^2=a_{n+1}- a_2 +(n-1) $

$両辺に a_1^2 を加えて$
\begin{eqnarray*} a_1^2+a_2 ^2 +\cdots a_n ^2&=&a_{n+1}- a_2 +(n-1)+a_1 ^2\\ &=& a_{n+1}- 2 +(n-1)+9\\ &=& a_{n+1}+n+6\\ \end{eqnarray*} \[\sum _{i=1}^n a_i ^2=a_1a_2 \cdots a_n +100 \ \ だから\] $\qquad a_1a_2 \cdots a_n +100 =a_{n+1}+n+6 $

$(1)より a_1a_2 \cdots a_n =a_{n+1}+1 $

$よって$
$\qquad a_{n+1}+1+100= a_{n+1}+n+6 $
$\qquad \therefore n=95$


(1)の別解


$まず、a_n > 1 \ \ を数学的帰納法で証明します。$
(i)$n=1 のとき a_1=3 \ \ だから成りたつ。$
(ii)$n=k のとき 成りたつとすると a_k > 1$
$このとき$
\begin{eqnarray*} a_{k+1} &=&a_k ^2 + a_k -1\\ &=&(a_k-1) ^2 +3(a_k-1)+1\\ &>&1 \end{eqnarray*} $よって n=k+1 のときも成りたつ。$

(i)(ii)$より すべての自然数nで a_n > 1$

$したがって$
$\quad a_{n+1}=a_n ^2 + a_n -1 \ \ より$
$\quad a_{n+1}+1=a_n ^2 + a_n =(a_n+1)a_n$

$\quad a_n+1 >2 (\ne 0) \ \ だから \quad a_n=\cfrac{a_{n+1}+1}{a_n+1}$

$nに2,3,\cdots を代入して、すべての積をとると$
\begin{eqnarray*} a_2a_3\cdots a_n &=&\cfrac{a_3+1}{a_2+1} \times \cfrac{a_4+1}{a_3+1} \times \cdots \times \cfrac{a_{n+1}+1}{a_n+1}\\ &=&\cfrac{a_{n+1}+1}{a_2+1}\\ &=&\cfrac{a_{n+1}+1}{3}\\ &=&\cfrac{a_{n+1}+1}{a_1}\\ \end{eqnarray*}
$分母を払って \quad a_1a_2\cdots a_n=a_{n+1}+1$

$\qquad \therefore a_{n+1}=a_1a_2\cdots a_n-1$


漸化式の拡張


$与えられた漸化式 a_{n+1}=a_n ^2 + a_n -1 \ \ を$

$\qquad a_{n+1}=a_n ^2 + p a_n -p  \hspace{15em}(1)$

$のように拡張します。$

$別解の解き方にならって$
$\qquad a_{n+1}+p=(a_n+p)a_n$

$\qquad a_n+p \ne 0 \ \ ならば(そのようなpとa_1を選んで)$

$\qquad a_n=\cfrac{a_{n+1}+p}{a_n+p}$
$すると$
\begin{eqnarray*} a_2a_3\cdots a_n &=&\cfrac{a_3+p}{a_2+p} \times \cfrac{a_4+p}{a_3+p} \times \cdots \times \cfrac{a_{n+1}+p}{a_n+p}\\ &=&\cfrac{a_{n+1}+p}{a_2+p}\\ \end{eqnarray*} $ここで$

$\qquad a_2+p=a_1 \hspace{20em}(2)$

$とおく。分母を払って$
$\qquad a_1a_2\cdots a_n=a_{n+1}+p$

$\qquad \therefore a_{n+1}=a_1a_2\cdots a_n-p \hspace{15em}(3)$

$このように設問のような式が導けました。$

$(3)を導くためには(2)が必要ですが$
$(2)より a_2=a_1-p$
$(1)に代入して$
$\qquad a_2=a_1 ^2 + p a_1 -p$
$\qquad a_1-p=a_1 ^2 + p a_1 -p$

$a_1 \ne 0 \ \ ならば \ \  a_1 でわって \ \ a_1=1-p \quad よって a_2=1-2p$

$つまり pを与えて a_1=1 -p ,\quad a_2=1-2p \ \ と定めると \ \ n \geqq 2 \ \ は必要なくなりすっきりする。$
$具体的にpにいくつかの値を入れてみましょう。$

(i)$\ \ P=-1 \ \ のときは$
$\qquad a_1=1-(-1)=2 ,\quad a_{n+1}=a_n ^2 -a_n +1 \ \ として数列\{a_n\}を定めるとき$
$\qquad a_{n+1}=a_1a_2 \cdots a_n +1 \ \ が成り立つことを証明せよ。$

(ii)$\ \ P=-2 \ \ のときは$
$\qquad a_1=1-(-2)=3 ,\quad a_{n+1}=a_n ^2 - 2a_n +2 \ \ として数列\{a_n\}を定めるとき$
$\qquad a_{n+1}=a_1a_2 \cdots a_n +2 \ \ が成り立つことを証明せよ。$

$などいくらでも考えられます。$

$設問はp=1 の場合ですが、このときは$
$\qquad a_1=1-p=1-1=0,\quad a_2=1-2p=1-2=-1$
$\qquad a_{n+1}=a_1a_2 \cdots a_n -1=-1$

$となって、全くおもしろくない結果になってしまいます。$

$そこで、あえて \ \ a_1=3,\quad a_2 \ は \ \ a_2+1=a_1 \ \ を満たすように2とし$

$\quad n \geqq 2 \ で、a_{n+1}=a_n ^2 + a_n -1 \ \ としたと考えられます。$

$このように数学的にはかなり無理のある設定となっていますが、では、なぜこのような設定にしたのでしょうか。$
$おそらく「ワナ」をしかけたかったからでしょう。$

$(2)で$
$\qquad a_{n+1}=a_n ^2 + a_n -1 \ \ より$
$\qquad a_n ^2 =a_{n+1}- a_n +1 $

$nに1,2,\cdots を代入して$

$\qquad a_1 ^2 =a_2- a_1 +1 $
$\qquad a_2 ^2 =a_3- a_2 +1 $
$\qquad \vdots $
$\qquad a_n ^2 =a_{n+1}- a_n +1 $

$これらを加えると$

$\qquad a_1^2+a_2 ^2 +\cdots a_n ^2=a_{n+1}- a_1 +n =a_{n+1}- 3+n$

$とするミス(ワナ)を誘いたかったのかもしれません。$
$もしそうだとすると、あまりいい気持ちはしません。$



ページの先頭へ↑



メインメニュー に戻る